2003 AIME II Problems/Problem 14

Revision as of 16:28, 5 January 2010 by Haoye (talk | contribs)

Problem

Let $A = (0,0)$ and $B = (b,2)$ be points on the coordinate plane. Let $ABCDEF$ be a convex equilateral hexagon such that $\angle FAB = 120^\circ,$ $\overline{AB}\parallel \overline{DE},$ $\overline{BC}\parallel \overline{EF,}$ $\overline{CD}\parallel \overline{FA},$ and the y-coordinates of its vertices are distinct elements of the set $\{0,2,4,6,8,10\}.$ The area of the hexagon can be written in the form $m\sqrt {n},$ where $m$ and $n$ are positive integers and n is not divisible by the square of any prime. Find $m + n.$

Solution

The y-coordinate of $F$ must be $4$. All other cases yield non-convex hexagons, which violate the problem statement.

Letting $F = (f,4)$, and knowing that $\angle FAB = 120^\circ$, we can use rewrite $F$ using complex numbers: $f + 4 i = (b + 2 i)\left(e^{(2 \pi / 3)}\right) = (b + 2 i)\left(-1/2 + \frac{\sqrt{3}}{2} i\right) = -\frac{b}{2}-\sqrt{3}+\left(\frac{b\sqrt{3}}{2}-1\right)i$. We solve for $b$ and $f$ and find that $F = \left(\frac{8}{\sqrt{3}}, 4\right)$ and that $B = \left(\frac{10}{\sqrt{3}}, 2\right)$.

The area of the hexagon can then be found as the sum of the areas of two congruent triangles ($EFA$ and $BCD$, with height $8$ and base $\frac{8}{\sqrt{3}}$ and a parallelogram ($ABDE$, with height $8$ and base $\frac{10}{\sqrt{3}}$.

$A = 2 \times \frac{1}{2} \times 8 \times \frac{8}{\sqrt{3}} + 8 \times \frac{10}{\sqrt{3}} = \frac{144}{\sqrt{3}} = 48\sqrt{3}.

Thus,$ (Error compiling LaTeX. Unknown error_msg)m+n = \boxed{51}$.

== Solution (Incomplete/incorrect) == {{image}}

From this image, we can see that the y-coordinate of F is 4, and from this, we can gather that the coordinates of E, D, and C, respectively, are 8, 10, and 6.

{{image}}

In this image, we have drawn perpendiculars to the$ (Error compiling LaTeX. Unknown error_msg)x$-axis from F and B, and have labeled the angle between the$x$-axis and segment$AB$$ (Error compiling LaTeX. Unknown error_msg)x$. Thus, the angle between the$x$-axis and segment$AF$is$60-x$so,$\sin{(60-x)}=2\sin{x}$. Expanding, we have

<center>$ (Error compiling LaTeX. Unknown error_msg)\sin{60}\cos{x}-\cos{60}\sin{x}=\frac{\sqrt{3}\cos{x}}{2}-\frac{\sin{x}}{2}=2\sin{x}$</center>

Isolating$ (Error compiling LaTeX. Unknown error_msg)\sin{x}$we see that$\frac{\sqrt{3}\cos{x}}{2}=\frac{5\sin{x}}{2}$, or$\cos{x}=\frac{5}{\sqrt{3}}\sin{x}$. Using the fact that$\sin^2{x}+\cos^2{x}=1$, we have$\frac{28}{3}\sin^2{x}=1$, or$\sin{x}=\sqrt{\frac{3}{28}}$. Letting the side length of the hexagon be$y$, we have$\frac{2}{y}=\sqrt{\frac{3}{28}}$. After simplification we see that$y=\frac{4\sqrt{21}}{3}$.

'''The following is incorrect as the hexagon is NOT regular (although it is equilateral). The previous work IS correct, so I am leaving it as part of an incomplete solution'''

The area of the hexagon is$ (Error compiling LaTeX. Unknown error_msg)\frac{3y^2\sqrt{3}}{2}$, so the area of the hexagon is$\frac{3*\frac{16*21}{3^2}*\sqrt{3}}{2}=56\sqrt{3}$, or$m+n=\boxed{059}$.

See also

2003 AIME II (ProblemsAnswer KeyResources)
Preceded by
Problem 13
Followed by
Problem 15
1 2 3 4 5 6 7 8 9 10 11 12 13 14 15
All AIME Problems and Solutions